Miranda is making bracelets. She already has 15 and she can make 3
per day. Determine the slope and the y-intercept.

Answers

Answer 1

Answer:

y=3x+15

Step-by-step explanation:

she can make 3 a day so 3x and she already has 15 so the line crosses the y-axis  at 15


Related Questions

How do I find a function from a graph? please I need help, I do not understand how this works. The picture below is my homework I am not asking for the answer I would just like someone to explain how this works please.

Answers

Answer:

F(x)=-(x+4)(x-1)

Step-by-step explanation:

the thing hit both -4 and 1 but in the function -4 become 4 and 1 become-1 so (x+4)(x-1) but we need to make it negative so -(x+4)(x-1)

Julia owns a food truck that sells tacos and burritos. She only has enough supplies to make 67 tacos or burritos. She sells each taco for $3.75 and each burrito for $6. Julia must sell no less than $330 worth of tacos and burritos each day. If 42 burritos were sold, determine all possible values for the number of tacos that Julia must sell in order to meet the requirements. Your answer should be a comma separated list of values. If there are no possible solutions, submit an empty answer.

Answers

The point would be the solution (25,40).

Let

The number of tacos sold is represented by x.

x denotes the quantity of burritos sold.

We are aware of this,

⇒ x + y 62

⇒ 3.75x + 6y = 330

The triangle shaded region is the answer

(25,40)

The point is one possible solution (25,40).

Remember that if an ordered pair is a solution to a system of inequalities, then the ordered pair must reside on the solution's region.

That indicates there were 30 tacos sold and 30 burritos sold.

Learn more about inequality at

https://brainly.com/question/25977724

#SPJ1

Examine the pattern of socks Above:

If the pattern continues, how many socks would b in figure 5?


(A.) 29

(B.) 41

(C.) 50

(D.) 52

Answers

(B.) 41

Visualizing it by drawing out boxes with how many rows and columns of socks there are, you'll see that by fig. 5, it's going to have 41 socks.

Identify the pair of numbers that represents the boiling point of water in degrees Fahrenheit and the boiling point of water in degree Celsius

Answers

Let's begin by identifying key information given to us:

The boiling point of water is 100 degrees Celsius

We will convert this temperature to Fahrenheit to obtain its equivalent using the formula:

[tex]\begin{gathered} F=\frac{9}{5}C+32_{} \\ C=100^{\circ} \\ F=\frac{9}{5}(100)+32=180+32 \\ F=212^{\circ} \end{gathered}[/tex]

Therefore, the first option is the correct answer (212 degrees Fahrenheit & 100 degrees Celsius)

find the measures for angles one and two

Answers

The measure of the angles x and y are 71 degrees and 109 degrees after applying the concept of vertically opposite angles.

What are vertically opposite angles?

It is defined as the angles when two lines intersect each other and at the intersecting point, some pair of angles are formed which we call vertically opposite angles, as the name describes that they have vertically opposite angles.

From the figure:

The value of y can be found by applying the concept of vertically opposite angles:

y =  109 degrees

x = [360 - 2(109)]/2

x = (360 - 218)/2

x = 71 degrees

Thus, the measure of the angles x and y are 71 degrees and 109 degrees after applying the concept of vertically opposite angles.

Learn more about the vertically opposite angles here:

brainly.com/question/24287162

#SPJ1

(PLEASE ANSWER THIS!!! I NEED THE ANSWER ASAP!!!) How many numbers are in this equation: 5, 7, 9 . . . 39, 41

Answers

The number of terms in the sequence is 19

How to determine the number of terms in the sequence?

The sequence is given as

5, 7, 9 . . . 39, 41

In the above sequence, we can see that

Type of sequence: Arithmetic sequenceFirst term, a = 5Last term, Tn = 41Common difference, d = 2

The nth term of an arithmetic sequence is

Tn = a + (n - 1)d

So, we have

5 + (n - 1) * 2 = 41

This gives

(n - 1) * 2 = 36

Divide by 2

n - 1 = 18

Add 1 to both sides

n = 19

Hence, there are 19 numbers in the sequence

Read more about sequence at

https://brainly.com/question/7882626

#SPJ1

Please help me please I’m begging I WILL GIVE YOU BRAINLEST

Answers

Answer:

x+3=2x-61 (corresponding angles)

3+61=2x-x

64=x

x=64

Answer:

x = 64

steps are in the photo

a graph shows a proportional relationship. what is the constant of proportionality

Answers

The constant of proportionality is known as the slope of the line.In the graph constant of proportionality is 1/2

What is constant of proportionality?

The constant of proportionality is known as the slope of the line.

Constant of proportionality is nothing but slope of a line

The formula for calculating slope is expressed as:

m = y₂-y₁/x₂-x₁

Given the coordinate points (2/3, 3/4 ) and (0,0).

y₂=0, y₁=3/4, x₂=0,x₁ =2/3

substitute the values of  y₂=0, y₁=3/4, x₂=0,x₁ =2/3 in slope formula to get constant of proportionality.

The constant of proportionality is expressed as:

m = 3/4 × 2/3

m =6/12

m=1/2

Hence the constant of proportionality is 1/2

Learn more on constant of proportionality:

brainly.com/question/1835116

#SPJ1

Rahul solved the equation 2(x – ) – 2 left-parenthesis x minus StartFraction 1 Over 8 EndFraction right-parenthesis minus StartFraction 3 Over 5 EndFraction x equals StartFraction 55 Over 4 EndFraction x = 2 left-parenthesis x minus StartFraction 1 Over 8 EndFraction right-parenthesis minus StartFraction 3 Over 5 EndFraction x equals StartFraction 55 Over 4 EndFraction . In which step did he use the addition property of equality?

A table titled Rahul's Solution with 2 columns and 5 rows. The first column, Steps, has the entries 1, 2, 3, 4. The second column, Resulting equations, has the entries, 2 x minus StartFraction 1 Over 4 EndFraction minus StartFraction 3 Over 5 EndFraction x equals StartFraction 55 Over 4 EndFraction, StartFraction 7 Over 5 EndFraction x minus StartFraction 1 Over 4 EndFraction equals StartFraction 55 Over 4 EndFraction, StartFraction 7 Over 5 EndFraction x equals StartFraction 56 Over 4 EndFraction, x equals 10.

Step 1
Step 2
Step 3
Step 4
Mark this and return

Answers

In step 2 addition property is used.

2x + (1 ÷ 4) - (3 ÷ 5)x = 55 ÷ 4

Simplifying x,

2x - (3 ÷ 5)x - (1 ÷ 4) = 55 ÷ 4

(10x - 3) ÷ 5 - (1 ÷ 4) = 55 ÷ 4

(7x ÷ 5) - (1 ÷ 4) = 55 ÷ 4

Add (1 ÷ 4) on both sides,

(7x ÷ 5) - (1 ÷ 4) + (1 ÷ 4) = 55 ÷ 4 + (1 ÷ 4)

(7x ÷ 5) = (55 + 1) ÷ 4

(7x ÷ 5) = 56 ÷ 4

Multiplying (5 ÷ 7) on both sides,

(7x ÷ 5) × (5 ÷ 7) = (56 ÷ 4) × (5 ÷ 7)

x = 10

In step 2 - Add (1 ÷ 4) on both sides, the addition property is being used.

Learn more about addition property at

https://brainly.com/question/14746969?referrer=searchResults

#SPJ1

Answer:

its c step three

Step-by-step explanation:

40 POINTS PLEASE ASAP!!!

Answers

It’s c because if you they don’t have the same slope on each of them.

Point M is on line segment
L
N

LN
. Given
L
N
=
4
x
,
LN=4x,
M
N
=
x
,
MN=x, and
L
M
=
3
,
LM=3, determine the numerical length of
L
N

.
LN
.

Answers

4 is the numerical length of line segment LN.

What is the numerical length of segment LN?

A line segment is simply the part of a line that connects two points or is bounded by two points.

Given the data in the question;

Point M is on line segment LNSegment LN = 4xSegment MN = xSegment LM = 3Numerical length of segment LN = ?

Since point M is on line segment LN, segment LN equals to the sum of segment LM and MN.

Hence;

Segment LN = segment LM + segment MN

4x = 3 + x

Solve for x

4x - x = 3

3x = 3

x = 3/3

x = 1

Now, we find the numerical length of segment LN;

Segment LN = 4x

Plug in x = 1 and simplify

Segment LN = 4( 1 )

Segment LN = 4

Therefore, the length of segment LN is 4.

Learn to solve more equations here: https://brainly.com/question/9236233

#SPJ1

if x= 10.069 what does x + 420=?

Answers

Answer:

10.069 +420 =430.069

Step-by-step explanation:

first step is to add the number 420 to the number before the decimal so 420+10 which equals to 430

Then

u add the number that comes after the decinal together or if only one of them has a number after the decimal place back that number after youre given answer....

Kaltlin has a patio that is in the shape of a rectangle with an area of 1500 ft?. She has decided to expand the patio so that it can hold mpatio will be a larger rectangle. The current patio has a length of 50 ft and a width of 30 ft. She plans on making the new length 2 timesthe new width 2 times the current width.

Answers

It is given that area of rectangle of Kaitlin is 1500 sq.ft. with length 50 ft and a width of 30 ft.

The new area when new length 2 times the current length and the new width 2 times the current width.

[tex]A=(50\times2)\times(30\times2)=100\times60=6000ft^2[/tex]

a. The size of Kaitlin new patio is

[tex]100\times60=6000ft^2[/tex]

b. The area of new patio is 4 times the area of current patio.

[tex]1500\times4=6000ft^2[/tex]

Kevin ran 4 miles more than Steve ran. The sum of their distances is 26 miles. How far did Steve run? The domain of the solution is (0, 4, 11, 22}
a. Define your variable.
b. Write algebraic expressions for the distances each person ran.
c. Write an equation that can help you solve the problem.
d. Solve the equation.
e. Check your answer.

Help Please write it simple

Answers

Using a system of equations, it is found that:

b) The domain of the solution is that Steve ran 11 miles.

c) Variable S gives the distance ran by Steve.

d) The algebraic expressions are:  K = 4 + S, K + S = 26.

e) The equation is: 4 + S + S = 26.

What is a system of equations?

A system of equations is when multiple variables are related, and equations are built to find the numeric values of each variable, according to the relations built in the problem.

In the context of this situation, the variables are given as follows:

Variable K: Distance ran by Kevin.

Variable S: Distance ran by Steve.

Kevin ran 4 miles more than Steve ran, hence:

K = 4 + S.

The sum of their distances is 26 miles, hence:

K + S = 26.

Replacing the first equation K = 4 + S on the second, we have that:

4 + S + S = 26

2S = 22

S = 11 miles.

Use the equation below to find P, if a = 5, b= 10, and c = 12

P = a + b + c​

Answers

Answer: The answer is 27

Step-by-step explanation:

if a=5 and b=10 and c=12 then just put the numbers in place of the letters so that's going to be :

P=5+10+12
P=15+12
P=27

Which number line shows the solution set for startabsolutevalue h minus 3 endabsolutevalue less-than-or-equal-to 5? a number line going from negative 8 to positive 8. closed circles are at negative 2 and positive 8. everything to the left of negative 2 and to the right of positive 8 is shaded. a number line going from negative 8 to positive 8. closed circles are at negative 2 and positive 8. everything between the points is shaded. a number line going from negative 8 to positive 8. open circles are at negative 2 and positive 8. everything between the points is shaded. a number line going from negative 8 to positive 8. open circles are at negative 2 and positive 8. everything to the left of negative 2 and to the right of positive 8 is shaded.

Answers

closed circles are at negative 2 and positive 8. everything between the points is shaded.

We have the equation:

|h - 3| [tex]\leq[/tex] 5

Remember that the equation:

|f(x)| = A

with A > 0

means that:

f(x) = A

or

f(x) = -A

Then for our case, we can rewrite:

|h - 3| [tex]\leq[/tex] 5

as:

(h - 3) [tex]\leq[/tex] 5

or

-(h - 3) [tex]\leq[/tex] 5

Now we can solve these two equations to find the two possible values of p.

From the first one, we get:

h [tex]\leq[/tex] 8

Now from the other equation, we can get the other solution:

h [tex]\geq[/tex] -2

Then the correct option is:

closed circles are at negative 2 and positive 8. everything between the points is shaded.

learn more about of circle here

https://brainly.com/question/24283506

#SPJ4

A soccer Practice is 90 minutes long, write an
expression for the number of minutes left after
m minutes have elapsed.

Answers

Answer:

Given that the information,

Length of a  soccer practice = 90 minutes

Time that has elapsed = m minutes

Let "y" be the minutes left

Expression for the number of minutes left is

[tex]y = (90 - m) min[/tex]

Which is true about the possible solutions of this function?

Answers

The possible solutions for the given function are as follows:

1 real root and 2 complex roots.

What is a function?Each member of X receives exactly one element of Y in the case of a function from one set to the other.The sets X and Y collectively constitute the domain and codomain of the function.Any assertion, rule, or piece of legislation that proves the causal link between the independent and dependent variables (the dependent variable).

In the graph,  the real roots are the values when it crosses the x-axis.

The function has one real root because it crosses the x-axis just once in this instance.The function has two complex roots since both of its ends go to infinity.

Therefore, the possible solutions for the given function are as follows:

1 real root and 2 complex roots.

To know more about the function, click on the link:

brainly.com/question/25638609

#SPJ13

a delivery company requires that any box delivered must have a length plus girth​ (distance around) totaling no more than 120 inches. find the dimensions of the box with maximum volume that can be sent.

Answers

the dimensions of the box is length 20 and width 40inches with maximum volume that can be sent. is V = 16000 inches ^3

length + girth=116 inches

w + 4x= 120

volume=x^2*w

w=120-4x

V=x^2* (120-4x)

for max value, dv/dx=0

dv/dx= 240x-12x^2

240x-12x^2=0

20-x=0

x=20

w=120-4(20)

=40

V=20^2*40==16000inches^3

learn more about of volume here

https://brainly.com/question/15243454

#SPJ4

The team needs new uniforms. The students plan to sell plush toy s ​(the school​ mascot) for​ $5 each. The students find three companies​ on-line that sell stuffed mascots. Company A sells s for ​$. Company B sells s for ​$. Company C charges ​$ for s. Which company has the best​ buy?

Answers

Company B has the best buy with a selling price of $2.79

Given: Company A sells 16 lions for $44.32

Company B sells 18 lions for $50.22

Company C sells 12 lions for $ 32.88

To find : The selling price of each lion

By using unitary method we get,

Company A sells 1 lion for $ 2.77

Company B sells 1 lion for $2.79

Company C sells 1 lion for $ 2.74

From the above statistics it is clear that Company B has the best buy .

To know more about selling price visit:

https://brainly.com/question/28017453

#SPJ9

Hi I need help with this question, please and thank you

Answers

We have the following number, we need to simplify:

[tex]6\cdot\sqrt[]{-2401}[/tex]

we can also express this as:

[tex]\begin{gathered} 6\cdot\sqrt[]{(-1)\cdot(2401)} \\ 6\sqrt[]{(-1)\cdot(49)\cdot(49)} \end{gathered}[/tex][tex]\begin{gathered} =6\sqrt[]{-1\cdot49^2} \\ =6\cdot\sqrt[]{-1}\cdot\sqrt[]{49^2} \\ =6\cdot\sqrt[]{-1}\cdot49 \\ =6\cdot49\cdot\sqrt[]{-1} \end{gathered}[/tex]

since:

[tex]\sqrt[]{-1}=i[/tex]

which represents the imaginary number,

then, we get:

[tex]6\sqrt[]{-2401}=6\cdot49\cdot i=294i[/tex]

Write a quadratic function in standard form whose graph has the given characteristics.
passes through (-8, 0), (-6. - 16), and (2, 0)
y =

Answers

Answer:

Step-by-step explanation:

HELP PLS Write the equation of the line in fully simplified slope-intercept form.

Answers

The equation of the line that passes through the x-intercept ( -2,0 ) and y-intercept ( 0,1 ) is y = (1/2)x + 1.

What is the equation of line of the graph?

The formula for equation of line is expressed as;

y = mx + b

Where m is slope and b is y-intercept.

From the graph, the x and y-intercept are; ( -2,0) and (0,1 )

Point 1 ( -2,0 )

x₁ = -2y₁ = 0

Point 2( 0,1 )

x₂ = 0y₂ = 1

First, we determine the slope of the line.

Slope m = y₂ - y₁ / x₂ - x₁

Slope m = 1 - 0 / 0 - (-2)

Slope m = 1 / 0 +2

Slope m = 1/2

Next, determine the y-intercept b by substituting the slope and a point into the equation of line formula.

y = mx + b

0 = (1/2)(-2) + b

0 = -2/2 + b

0 = -1 + b

b = 1

Now that the values of slope m and y-intercept b is known, plug these into y = mx + b to determine the equation of the line.

y = mx + b

y = (1/2)x + 1

Therefore, the equation of the line is y = (1/2)x + 1.

Learn more about equation of line here: brainly.com/question/2564656

#SPJ1

23. According to the Guinness Book of World Records, the world’s smallest horse is Thumbelina. Thumbelina is 42.5 cm tall and eats about 0.3 kg of food per day. A former world record holder ate food in the same proportion to its height. If it was 46.25 cm tall, how much did it eat? Give your answer to the nearest hundredth of a kilogram.

Answers

The food that the former world record holder should eat per day is 0.33kg.

How much should should the horse eat?

The first step is to determine the ratio of the smallest horse's food per day to the height of the horse. This can be determined by dividing the food eaten per day by the height of the horse.

Division is the operation used in mathematics that is used to determine the quotient of two or more numbers

Ratio of the height and food eaten per day = 0.3 / 42.5

The next step is to multiply the ratio gotten in the previous step by the height of the former world holder

Food the former holder should eat = (0.3 / 42.5) x 46.25 = 0.33 kg

To learn more about ratios, please check: https://brainly.com/question/11911538

#SPJ1

Prove that: [tex](a+b)^{-1} . (a^{-1} + b^{-1} ) = (ab)^{-1}[/tex]

Answers

Step-by-step explanation:

[tex] \frac{1}{a + b} \times ( \frac{a + b}{ab}) = \frac{1}{ab} [/tex]

Answer with step-by-step explanation:

[tex](a+b)^-^1*(a^-^1+b^-^1)=(ab)^-^1[/tex]

First, convert these into positive indices.

[tex]\frac{1}{(a+b)} *(\frac{1}{a} +\frac{1}{b})=\frac{1}{ab}[/tex]

And now, let us solve the left side.

[tex]\frac{1}{(a+b)} *(\frac{1}{a} +\frac{1}{b})\\\\[/tex]

First, solve the brackets. That is add the fractions inside the brackets.

[tex]\frac{1}{(a+b)} *(\frac{1}{a} +\frac{1}{b})\\\\\frac{1}{(a+b)}*(\frac{1*b}{a*b} +\frac{1*a}{b*a})\\\\\frac{1}{(a+b)}*(\frac{b}{ab} +\frac{a}{ab})\\\\\frac{1}{(a+b)}*\frac{(a+b)}{ab}[/tex]

Now multiply the fractions.

[tex]\frac{1}{ab}[/tex]

So, it's clear that the left side equals the right side.

Left side = Right side

[tex]\frac{1}{ab}=\frac{1}{ab}[/tex]

∴ [tex](a+b)^-^1*(a^-^1+b^-^1)=(ab)^-^1[/tex]

0.5s < 20

Solve the inequality

Answers

Answer:

x ∠ 40

Step-by-step explanation:

0.5x ∠ 20

Divide both sides by 0.5:

x ∠ 40

X: 3 6 9 12 y: -22 -43 -64 -85 point-slope form:_ slope-intercept form:​

Answers

point slope form = y-(-43)=[tex]-\frac{2}{21}[/tex](x-3)

slope intercept form = -[tex]\frac{152}{7}[/tex]=c

What is slope?

Finding the ratio of "vertical change" to "horizontal change" between any two unique locations on a line yields the slope. Occasionally, the ratio is written as a quotient (also known as a "rise over run"), which produces the same number for every two unique points on the same line. Negative "rise" refers to a diminishing line. The line could be functional, established by a road surveyor, or depicted in a graphic that represents a road or a roof as a description or a design.

The absolute value of the slope is used to determine how steep, incline, or grade a line is. The steeper the line, the larger the absolute magnitude of the slope. A line's direction might be either horizontal, ascending, decreasing, or vertical

If a line is rising from left to right, it is increasing. Since m>0, the slope is positive.

If a line decreases from left to right, it is said to be diminishing. The slope is inverse, or m0.

A line has zero slope if it is horizontal. This function has a fixed value.

A line's slope is ambiguous if it is vertical.

Slope= [tex]\frac{x-x1}{y-y1}[/tex]= [tex]\frac{3-5}{-22+43}[/tex]=[tex]\frac{-2}{21}[/tex]

point slope form= y-(-43)=[tex]-\frac{2}{21}[/tex](x-3)

slope intercept formy=mx+c

-22=[tex]-\frac{2}{21}[/tex]*3+c

-22+[tex]\frac{2}{7}[/tex]=c

-[tex]\frac{152}{7}[/tex]=c

To know more about slope visit: https://brainly.com/question/3605446

#SPJ9

Subtract (-2)-(-1) using number line

Answers

Answer:

picture

Step-by-step explanation:

everything in the picture

i hope that this helps

Answer:

the answer is -1 .........


Evaluate. Write your answer as a whole number or as a simplified fraction.

Answers

Answer:

1 / 121

Step-by-step explanation:

4^0 = 1

11^2 = 121

4^0 / 11^2 = 1 / 121

The bank charged you an overdraft fee of $35 for not having enough money in your account.
What is the balance after the bank charged you the overdraft fee?

Answers

What was the balance before and if there wasn’t any money and they charged you it would be -35
Other Questions
Which of the following statements is INCORRECT?a) The Mayan civilization arose in the highland areas of Mexico.b) The Aztec civilization was founded after the Maya Civilization.c) The ceremonial center of the Aztec civilization, named Tenochtitlan, was located in the Valley of Mexico.d) Maya languages are still used today in parts of Mexico.e) The Aztecs were conquered by the Spanish. How does senator obamas use of the metaphor ""beacon of freedom"" reflect the purpose of inspiring the audience to take action in the upcoming election? select all that apply. 1) Choose whether thyroid or thymus applies to each of the following:WhiteDark redShaped like a VShaped like a small bean Need answer as soon as possible, emergency, please be 100% sure, thank you Help need please!!!!!!!factorize 36a-12b-60c The posterior axial muscle that crosses the glenohumeral joint is the __________. please help me out fr The quadrilateral RSTU is inscribed in the circle shown below. S R U Jake wants to prove the theorem that says that the measure of the quadrilateral's opposite angles add to 180. He knows that the measure of angle Ris half the measure of are STU and that the measure of angle T is half the measure of arc SRU. Which of the following is an appropriate step to prove that ZR + ZT =180? Kim buys 6 pounds of apples.What is the weight of the apples in kilograms?Round to the nearest tenth, if necessary. Let f(x) = (6x - 7) and g(x) = 6x - 7.Given that f(x) = (h g)(x), find h(x).0080Clear allEnter the correct answer.+?DONE Who pays the true cost of so many novice climbers trying to summit the mountain? What is that cost? An air traffic controller spots two planes flying at the same altitude. Their flight paths form a right angle at point P. One plane is 150 miles from point P and is moving at 440 miles per hour. The other plane is 200 miles from point P and is moving at 440 miles per hour. Write the distance s between the planes as a function of time t. Solve the trigonometric equation for all values 0 x < 2.tan x - 1 = 0 Who held the belief of predestination and what kind of lives did the followers of this belief live? Hint: first convert the given final speed (in mph) to find ft/s, then solve for the distance and acceleration. an organizations is a statement of its fundamental, unique purpose that sets a business apart from other firms of its type and identifies the scope of the businesss operations in product and market terms. What is the answer to this eqaution 7.68+3.1812 ater is considered hypoxic (lacking oxygen) if it contains 2 mg/l or less dissolved oxygen. think of the food chain that exists in the gulf of mexico: what are the types of organisms that likely live at the bottom of the gulf, where the water is most hypoxic? how can a lack of oxygen affect them? which cellular processes are affected by a lack of oxygen? using a lancet over 2.5mm to 3mm is acceptable when? What is 4/5 the sum of w and z given that w = 12 and z = 43